JEE Knocout Test QP_Solution_Dec22 (1)

download JEE Knocout Test QP_Solution_Dec22 (1)

of 76

Transcript of JEE Knocout Test QP_Solution_Dec22 (1)

  • 8/12/2019 JEE Knocout Test QP_Solution_Dec22 (1)

    1/76

    JEE Knockout Test Solution

    1. Time duration for this test is 180 minutes. This test consists of 90 questions. Themaximum marks are 360.

    2. There are three parts in the question paper, namely, Part A: Mathematics, Part B:

    Physics and Part C: Chemistry. Each question is awarded 4 (four) marks for correctresponse.

    3. One-fourth (1/4) marks will be deducted for indicating incorrect response of eachquestion. No deduction from the total score will be made if no response is indicated foran item in the answer sheet.

    4. This test contains 90 Multiple Choice Questions with single correct answer. EachQuestion has four choices (1), (2), (3) & (4), out of which Only One is Correct.

    5. Filling up more than one response in any question will be treated as wrong response andmarks for wrong response will be deducted accordingly as per the instruction 3 above.

    Part A

    Mathematics

    Question 1:If147 , then find .

    3 4

    iZ Z

    i

    (1) 27 (2) (2)7

    (3) (27)i (4) (27) i

    Chapter:Complex Numbers

    Level of Difficulty:EasySolution:We have

    7

    3 4

    iZ

    i

    Simplifying (i.e., rationalizing the denominator), we get

    7 3 4 21 4 28 3

    3 4 3 4 9 16

    25 25

    25

    i i i i

    i i

    i

    = 1i

    Therefore,

  • 8/12/2019 JEE Knocout Test QP_Solution_Dec22 (1)

    2/76

    1414

    2 7

    2 7

    7

    7(1 )

    3 4

    [(1 ) ]

    (1 2 )

    ( 2 )

    ii

    i

    i

    i i

    i

    Correct Option: (3).

    Question 2:If 4 4 10,Z Z then the difference between the maximum and the minimum values of

    Z is

    (1) 2 (2) 3

    (3) 41 5 (4) 0

    Chapter:Complex Numbers

    Level of Difficulty:Hard

    Solution

    4 4 10Z Z

    Ps+Ps'= 2a

    which implies that foci at 4 and4 and a= 5 as shown in the following figure.

    Now,

    b2= 25(1e

    2)

    = 25(5e)2

    = 2516 = 9

    b= 3

    Zlies on the ellipse circumference Z denotes the distance from the origin. Therefore,

    bi

    -bi

  • 8/12/2019 JEE Knocout Test QP_Solution_Dec22 (1)

    3/76

    max

    min

    5

    3

    Z

    Z

    Thus, the difference between the maximum and the minimum values of Z is

    max min 5 3 2Z Z

    Correct Option: (1).

    Question 3:If [x]27[x] + 10 < 0 and 4[y]216[y] + 7 < 0, then [x+y] cannot be ([] denotes greatest

    integer function)

    (1) 7 (2) 8

    (3) 9 (4) Both (2) and (3)

    Chapter:Quadratic Equations and Expressions

    Level of Difficulty:Moderate

    Solution: We have

    2[ ] 7[ ] 10 0x x

    ([ ] 5) ([ ] 2) 0x x

    2 [ ] 5x

    [ ] 3or4x

    [3, 5)x

    and

    24[ ] 16[ ] 7 0y y

    (2[ ] 7)(2[ ] 1) 0y y

    1 7[ ]

    2 2y

    [ ] 1 or 2 or 3y

    [1, 4)y

    Therefore,

    [4,9)

    [ ] {4,5,6,7,8}

    x y

    x y

    Hence, [x+y] cannot be 9.

    Correct Option:(3).

  • 8/12/2019 JEE Knocout Test QP_Solution_Dec22 (1)

    4/76

    Question 4:If a, band care the roots of the equationx3+ 2x2+ 1= 0, find .

    a b c

    b c a

    c a b

    (1) 8 (2)8

    (3) 0 (4) 2

    Chapter:Matrices, Determinants and System of Equations

    Level of Difficulty:Moderate

    Solution: As a, band care the roots ofx3+ 2x2+ 1 = 0, we have

    a+ b+ c=2

    ab+ bc+ ca= 0

    abc=1

    Now, for finding the value of

    a b c

    b c ac a b

    , evaluating using first row, we get

    a(bca2)b(b2ac) +c(abc2) = abca3b3+ abc+ abcc3

    = 3abca3b3c3

    =(a3+ b

    3+ c

    33abc)

    =(a+ b+ c) (a2+ b2+ c2abbcca)

    =(2) [(2)23(0)] = 8

    Correct Option:(1).

    Question 5:If1 2 cos sin

    , ,0 1 sin cos

    TA P Q P AP

    , find 2014 .TPQ P

    (1)20141 2

    0 1

    (2)1 4028

    0 1

    (3)2013 2014 2013( )TP A P (4) 2014TP A P

    Chapter:Matrices, Determinants and System of Equations

    Level of Difficulty:Hard

    Solution: If1 2 cos sin

    , , ,0 1 sin cos

    TA P Q P AP

    we have

    PQ20/4PT ( )( ) ( )

    2014 times

    T T T T P P AP P AP P AP P

    ( ) ( ) ( ) ( ) ( )T T T T T PP A PP A PP PP A PP

  • 8/12/2019 JEE Knocout Test QP_Solution_Dec22 (1)

    5/76

    Matrix multiplication is associative:

    cos sin cos sin

    sin cos sin cos

    TPP

    2

    1 0

    0 1I

    Hence,PQ2014PT=A2014

    21 2 1 2 1 2 1 4

    0 1 0 1 0 1 0 1A A

    3 1 4 1 2 1 6

    0 1 0 1 0 1A

    4 1 6 1 2 1 8

    0 1 0 1 0 1A

    20141 2 1 4028and =0 1 0 1

    n nA A

    Correct Option:(2).

    Question 6:Find the value of3 3 3 3 3 3 3 3 3 3

    2 6 12 20

    1 1 2 1 2 3 1 2 3 4

    up to 60 terms.

    (1) 2 (2)1

    2

    (3) 4 (4)1

    4

    Chapter:Progressions, Sequences and Series

    Level of Difficulty:Moderate

    Solution

    3 3 3 3 3 3 3 3 3 3

    2 6 12 20...

    1 1 2 1 2 3 1 2 3 4

    3 3 3 3 3 3

    1 2 2 3 3 4...

    1 1 2 1 2 3

    3 3 31

    ( 1)lim

    1 2 ...

    n

    n

    n n

    n

    21

    ( 1)lim

    ( 1)

    2

    n

    n

    n n

    n n

  • 8/12/2019 JEE Knocout Test QP_Solution_Dec22 (1)

    6/76

    1

    1lim 4

    ( 1)

    n

    n n n

    1

    1 14 lim

    1

    n

    n n n

    1 1 1 1 1 14 lim ...

    1 2 2 3 1n n n

    4 lim 41n

    n

    n

    Correct Option: (3).

    Question 7:In the expansion of (1 +x)2(1 +y)3(1 +z)4(1 + w)5, the sum of the coefficient of the terms

    of degree 12 is

    (1) 61 (2) 71

    (3) 81 (4) 91

    Chapter:Binomial Theorem

    Level of Difficulty:Hard

    Solution

    (1 +x)2(1 +y)

    3(1 +z)

    4(1 + w)

    5

    General term 2 3 4 5 a b d ea b d eC C C C x

    2 3 4 5 14 14

    12 1212

    14 13

    or 2a b d ea b d e C C C C C C

    = 91

    Correct Option:(4).

    Question 8:Let Sbe the set of all functions from the set {1, 2, , 10} to itself. One function is selected

    from S, the probability that the selected function is oneone onto is

    (1)9

    9!

    10 (2)

    1

    10

    (3)100

    10! (4)

    10

    9!

    10

    Chapter: Probability

    Level of Difficulty:Moderate

    Solution:Letfbe function from {1, 2,,10} to itself total functions possible is 1010

    . The number of

    oneone onto functions possible is 10!

    Hence, the probability of selected function to be oneone onto is

  • 8/12/2019 JEE Knocout Test QP_Solution_Dec22 (1)

    7/76

    10 9

    10! 9!

    10 10

    Correct Option:(1).

    Question 9: Two friends visit a restaurant randomly during 5 pm to 6 pm. Among the two, whoever

    comes first waits for 15 min and then leaves. The probability that they meet is

    (1)1

    4 (2)

    1

    16

    (3)7

    16 (4)

    9

    16

    Chapter: Probability

    Level of Difficulty:Hard

    Solution: Let the friends come to the restaurant at 5 h x min and 5 h y min, respectively, where, [0,60]x y .

    Hence, the sample space consists of all points (x, y) lying in 60 60 square as shown above and for

    favourable cases, 15x y , that is, 15 xy 15 which is shown by shaded region in the graph

    shown below.

    Hence, the probability that they will meet is given by

    145 45

    2160 60

    23 7

    14 16

    Correct Option:(3).

  • 8/12/2019 JEE Knocout Test QP_Solution_Dec22 (1)

    8/76

    Question 10: If ( ) ( ) ( )r a m n b n l c l m and [ ] 4l mn , find .( )

    a b c

    r l m n

    (1)1

    4 (2)

    1

    2

    (3) 1 (4) 2

    Chapter: Vectors

    Level of Difficulty:Moderate

    Solution

    ( ) ( ) ( )r a m n b n l c l m

    where

    [ ] 4l mn

    4r l a

    4r m b

    4r n c

    which imply that

    1

    4( )

    a b c

    r l m n

    Correct Option

    The correct option is (1).

    Question 11: The volume of tetrahedron, for which three co-terminus edges are , anda b c , is k units.

    Then, the volume of a parallelepiped formed by , 2a b b c and 3a c is

    (1) 6k (2) 7k

    (3) 30k (4) 42k

    Chapter: Vectors

    Level of Difficulty:Hard

    Solution

    The volume tetrahedron is given by

    1[ ] [ ] 6

    6k a b c a b c k

    The volume of parallelepiped is given by

    [ 2 3 ]a b b c a c

    [ 2 3 ] [ 2 3 ]a b c ac b b c a c

  • 8/12/2019 JEE Knocout Test QP_Solution_Dec22 (1)

    9/76

    [ 3 ] [ 2 3 ] [ 3 ] [ 2 3 ]a b a c a c a c b b a c b c a c

    [ ] [ 2 3 ]a b c b c a

    [ ] 6[ ]a b c a b c

    7[ ]a b c

    Volume is 42k.

    Correct Option:(4).

    Question 12: Statement 1: Negation of the statement if price increases, then demand falls is price

    increases and demand does not fall

    Statement 2:Negative ofp qis ~p q .

    (1) Statement 1 is True; Statement 2 is True; Statement 2 is a correct explanation for Statement 1.

    (2) Statement 1 is True; Statement 2 is true; Statement 2 is not a correct explanation for Statement 1.

    (3) Statement 1 is True; Statement 2 is False.

    (4) Statement 1 is False; Statement 2 is True.

    Chapter: Mathematical Reasoning

    Level of Difficulty:Easy

    Solution: The negation ofp qis ~ .p q Hence, the negation of if price increases then demand falls

    is price increases and demand does not falls.

    Statement 1 is correct, but Statement 2 is incorrect.

    Correct Option:(3).

    Question 13: Simultaneously two dice are thrown five times. If getting a sum as multiple of 3 is

    considered as success, then the variance of the distribution of number of successes is

    (1)5

    3 (2)

    10

    3

    (3)10

    9 (4)

    140

    121

    Chapter: StatisticsLevel of Difficulty:Hard

    Solution

    The sum as multiple of 3, possibilities are 3, 6, 9 and 12, that is, (1, 2), (2, 1), (1, 5) (2, 4) (3, 3) (4, 2) (5,

    1) (3, 6) (4, 5) (5, 4) (6, 3) and (6, 6). There are 12 possibilities and hence the probability of success is

  • 8/12/2019 JEE Knocout Test QP_Solution_Dec22 (1)

    10/76

    12 1

    36 3

    It is a binomial distribution with n= 5 and1

    3p , which implies that the variance is

    1 2 1053 3 9

    npq

    Correct Option:(3).

    Question 14: If 270 < < 360, then find 2 2(1 cos ) .

    (1) 2sin4

    (2) 2sin

    4

    (3) 2sin

    4

    (4) 2cos

    4

    Chapter: Trigonometric Ratios and Identities

    Level of Difficulty:Moderate

    Solution: When, 270 < < 360, we have

    2(1 cos ) 22 2cos2

    which is non-negative. Now, the above equation can be written as

    2(1 cos ) 2 cos

    2

    2cos2

    cos 0 when135 180

    2 2

    Now, let us consider that

    2 2(1 cos )

    which is non-negative. That is,

    2

    2 2(1 cos ) 2 2cos2

    2 1 cos2

    2 2sin4

    2 sin4

  • 8/12/2019 JEE Knocout Test QP_Solution_Dec22 (1)

    11/76

    1352sin sin 0when 90

    4 4 2 4

    Correct Option:(2).

    Question 15: The number of solutions of sin1x+ sin1(1 +x) = cos 1xis/are

    (1) 0 (2) 1

    (3) 2 (4) infinite

    Chapter: Inverse Trigonometric Functions

    Level of Difficulty:Easy

    Solution

    sin1(1 +x) is defined forx< 0 and1 1sin cos 1 1.

    2x x x

    The given equation is1 1 1sin sin (1 ) cosx x x

    which can be written as

    1 1 1cos cos (1 ) cos2 2

    x x x

    1 1cos (1 ) 2cosx x

    1 1 2cos ( 1 ) 2 cos (2 1)x x

    1 1 2

    cos ( 1 ) cos (2 1) 2x x

    1 1 2cos ( 1 ) cos (2 1)x x

    21 2 1 1x x

    0x

    Which implies that the total number of solutions sin1x+ sin1(1 +x) = cos

    1xis only one.

    Correct Option:(2).

    Question 16: IfP(A) is the power set of setA, which of the following statements is false?

    (1) { : }A B x x A x B (2) A B B A

    (3) N I Q R C (4) ( ) { : }P A X X A

    Chapter: Sets

    Level of Difficulty:Easy

    Solution

  • 8/12/2019 JEE Knocout Test QP_Solution_Dec22 (1)

    12/76

    1. :x x A x B means that all elements exist in setAalso exist in setB. Similarly, A B means

    that all elements exist in set Aalso exist in set B. The Venn diagram can be drawn as shown in

    any of the following two figures:

    Therefore, option (1) is correct.

    2. A B meansAis a subset ofBorB A means thatBis a super set ofA. Therefore, option (2) is

    correct.

    3. It is a well-known fact that N I or N I and hence both are correct. Therefore, option (3) is

    correct.

    4. The statement ( ) { : }P A X X A is wrong and the correct statement is ( ) { : }P A X X A , that

    is, setX= setAis also included. Therefore, option (4) is incorrect or false.

    Correct Option:(4).

    Question 17: The value of2

    lim ( , , , , {0})x

    ax bx ca b c d e

    dx e

    depends on the sign of

    (1) aonly (2) donly

    (3) aand donly (4) a, band donly

    Chapter: Limits

    Level of Difficulty:Moderate

    Solution

    2 ( / )lim lim

    ( / )

    lim

    x x

    x

    ax bx c ax b c x

    dx e d e x

    a bx

    d d

    = +ifa

    d

    is positive.

    =ifa

    d

    is negative.

  • 8/12/2019 JEE Knocout Test QP_Solution_Dec22 (1)

    13/76

    Correct Option:(3).

    Alternate Solution

    2 2

    2

    ( / ) ( / )lim lim

    ( / ) ( / )x x

    ax bx c a b x c x

    dx e d x e x

    Here,2

    e d

    xx . Therefore,

    2

    lim lim/

    if 0 and 0if 0

    if 0 and 00

    if 0 and 0if 0

    0 if 0 and 0

    x x

    ax bx c a

    dx e d x

    a dad

    a d

    a a dd

    a d

    Question 18: Ify= (x3)(x2)(x1)x(x+ 1)(x+ 2)(x+ 3), then2

    2

    d y

    dxatx= 1 is

    (1) 101 (2) 48

    (3) 56 (4) 190

    Chapter: Differentiation

    Level of Difficulty:Easy

    Solution

    We have

    2 2 2( 9)( 4)( 1)y x x x x

    6 4 2( 14 (49) 36)x x x x

    7 5 314 49 36x x x x

    Therefore,

    6 4 27 70 147 36dy

    x x xdx

    Thus,

    2 5 3

    2 42 280 294d y x x x

    dx

    2

    2

    1

    42 280 294

    x

    d y

    dx

    = 56

    Correct Option:(3).

  • 8/12/2019 JEE Knocout Test QP_Solution_Dec22 (1)

    14/76

    Question 19: If sin cos( )2

    dy xx

    dx

    , thenyis strictly increasing in

    (1) (3, 4) (2)5 7

    ,2 2

    (3) (2, 3) (4)1 3

    ,2 2

    Chapter: Applications of Derivatives

    Level of Difficulty:Moderate

    Solution: Let us draw the graph of

    ( ) sin2

    f x x

    and

    ( ) cos( )g x x

    on the samexy-plane as shown in the following figure.

    From this graphical representation, it is clear that yis strictly increasing in

    5 7,

    2 2

    f(x)

    x

    y

  • 8/12/2019 JEE Knocout Test QP_Solution_Dec22 (1)

    15/76

    Because for all values ofx,

    5 7

    2 2x

    That is,

    sin 02

    x

    and

    cos ( ) 0x

    which imply that

    0dy

    dx

    which means thatyis strictly increasing.

    Correct Option:(2).

    Question 20: Given that

    2 1 1

    2 2

    2 3

    2 1(1 ) 2 1 1 1n n n

    ndx x dx

    nx n x x

    . Find the value of

    1

    40 2.

    1

    dx

    x

    (You may or may not use reduction formula given.)

    (1)11 5

    48 64

    (2)

    11 5

    48 32

    (3)1 5

    24 64

    (4)

    1 5

    96 32

    Chapter: Indefinite Integration

    Level of Difficulty:Moderate

    Solution

  • 8/12/2019 JEE Knocout Test QP_Solution_Dec22 (1)

    16/76

    3

    11 1

    4 2 4 10 0 2)20

    11

    3 2 2 2 20

    01

    1

    2 200

    1

    5

    62(4 1)(1 ) (11

    1 5 5 30 .

    6 6 46(2) 2(2)(1 ) (1 )

    1 5 1 5 5 10

    48 6 16 8 8 22(1)(1 ) 1

    1 5 5 1 50 tan

    48 6 16 8 4 16

    dx x dx

    x xx

    x dx

    x x

    x dx

    x x

    x

    1

    0

    7 5 50

    6 16 8 4 16 4

    22 5

    6 16 64

    11 5

    48 64

    Correct Option:(1).

    Alternate Solution

    1

    40 21

    dxI

    x

    Put tanx ; therefore, 2secdx d .

    2

    480

    sec

    sec

    d

    That is,

    /4 6

    0

    2/4

    0

    /4 /4 /42 2

    0 0 0

    /4 /4 /4

    0 0 0

    /4 /4

    0 0

    (cos )

    3cos cos3

    4

    9 1 3cos (cos3 ) cos cos3

    16 16 8

    4 cos 29 1 cos2 1 1 cos6 3

    16 2 16 2 8 2

    9 sin 2 1 sin 6

    32 2 16 2 6

    d

    d

    d d

    cosd d

    /4

    0

    3 sin 4 sin 2

    8 2 4 2

    9 1 1 1 3 10 0

    32 4 2 16 2 4 6 8 2 2

    5 11

    64 48

  • 8/12/2019 JEE Knocout Test QP_Solution_Dec22 (1)

    17/76

    Question 21: If the length of the normal for each point on a curve is equal to the radius vector, then the

    curve

    (1) is a circle passing through origin

    (2) is a circle having centre at origin and radius > 0

    (3) is a circle having centre onx-axis and touchingy-axis

    (4) is a circle having centre ony-axis and touchingx-axis

    Chapter: Differential Equations

    Level of Difficulty:Hard

    Solution: The length of the normal is

    2

    1 dy

    ydx

    The length of radius vector of a point (x,y) on the curve is xi yj , that is, 2 2 .x y It is given that

    2 2 21 ( ')x y y y

    Squaring on both sides of this equation, we get

    2 2 2 2

    22 2 2 2

    2

    2

    [1 ( ') ]

    or

    x y y y

    dyx y y y

    dx

    dyx y dx

    dy dyy x y x

    dx dx

    Now,

    dyy x

    dx

    ydy xdx

    Integrating on both sides, we get

    2 2

    2 2 2 2

    2 2

    2 or Constant

    y x c

    x y c x y

    This answer does not exist in the given options. So, consider the other alternative:

    y dy x dx

    Integrating on both sides, we get

  • 8/12/2019 JEE Knocout Test QP_Solution_Dec22 (1)

    18/76

    2 2

    2 2

    y xc

    2 2 Constantx y

    and this constant is > 0 in practical sense.

    Correct Option:(2).

    Question 22: Find the value of/4

    4

    0

    (sin ) .x dx

    (1)3

    16

    (2)

    3 1

    32 4

    (3)3 3

    32 4

    (4)

    3 7

    16 8

    Chapter: Definite Integration

    Level of Difficulty:Easy

    Solution

    We have

    /44

    0

    (sin )I x dx

    (1)

    We know that

    2 1 cos2sin2

    xx

    Therefore,

    24 4

    2

    1 cos2sin (sin )

    2

    1[1 2cos2 (cos2 ) ]

    4

    1 1 cos 41 2cos2

    4 2

    1 3 cos 42cos2

    4 2 2

    xx x

    x x

    xx

    xx

    Substituting this value of 4sin x in Eq. (1), we get

  • 8/12/2019 JEE Knocout Test QP_Solution_Dec22 (1)

    19/76

    /4

    0

    /4/4 /4

    0 00

    3 1 1cos 2 cos 4

    8 2 8

    3 1 1[sin 2 ] [sin 4 ]

    8 4 32

    3 1 1. (1 0) (0 0)8 4 4 32

    3 1

    32 4

    I x x dx

    x x x

    Correct Option:(2).

    Alternate Solution: We have

    /44

    0

    (sin )I x dx

    which can be written as

    2 2

    2 2 2

    2

    (sin ) (1 cos )

    1sin 4sin cos

    4

    1 cos 2 1(sin2 )

    2 4

    1 1 1 1 cos 4sin 2

    2 4 4 2

    sin 2 1 1sin 4

    2 4 8 32

    3 sin 2 sin 4

    8 4 32

    J x x dx

    xdx x x dx

    xdx x dx

    xx x dx

    x xx x c

    x x

    x c

    Using the given limits, the above equation becomes

    /4 /4 /4/4

    00 0 0

    3 sin 2 sin 4[ ]

    8 4 32

    3 1

    32 4

    x xI J x

    Question 23: The area of the loop formed by 2 3(1 )y x x dx is

    (1)1 4

    0x x dx

    (2)

    14

    0

    2 x x dx

  • 8/12/2019 JEE Knocout Test QP_Solution_Dec22 (1)

    20/76

    (3)1 4

    1x x dx

    (4)

    1/24

    0

    4 x x dx

    Chapter: Area Under the Curve

    Level of Difficulty:Moderate

    Solution: We have

    2 3(1 )y x x (1)

    For 1x , 2y is negative. Since the square of a real number cannot be negative, ydoes not exist at x =

    0 or atx= 1;y= 0. Let1

    2x . Therefore, from Eq. (1), we get

    2 1 1 712 8 16

    7

    4

    y

    y

    Also, forx< 0, 2y is negative. Therefore, the required area is

    1 13

    0 0

    14

    0

    2 2 ( ) 1

    2

    y dx x x dx

    x x dx

    Correct Option: (2).

    Quick Tip: Whenx< 0, no curve exists; whenx< 1, no curve exists. Therefore,y2cannot be negative ify

    is real.

    1

  • 8/12/2019 JEE Knocout Test QP_Solution_Dec22 (1)

    21/76

    Question 24: The equation of a plane passing through the line of intersection of the planes

    2 10 0x y z and 3 5x y z and passing through the origin is

    (1) 5 3 0x z (2) 5 3 0x z

    (3) 5 4 3 0x y z (4) 5 4 3 0x y z

    Chapter: Three-Dimensional Geometry

    Level of Difficulty:Moderate

    Solution

    We know that the equation of the plane passing through the line of intersection of planes 1 0p and

    2 0p is

    1 2 0p p

    That is,

    ( 2 10) (3 5) 0x y z x y z (1)

    Since, this plane passes through the origin, (0,0,0) satisfies this equation. This implies that

    ( 10) ( 5) 0

    2

    Substituting the value of in Eq. (1), we get

    ( 2 10) 2(3 5) 0x y z x y z

    That is,

    5 3 05 3 0x zx z

    Correct Option:(2).

    Question 25: Find the locus of a point whose distance from x-axis is twice the distance from the point

    (1, 1,2) .

    (1) 2 2 2 4 6 0y x y z (2) 2 2 2 4 6 0x x y z

    (3) 2 2 2 4 6 0x x y z (4) 2 2 2 4 6 0z x y z

    Chapter: Three-Dimensional Geometry

    Level of Difficulty: Hard

    Solution: Let the point ( , , )PP P

    P x y z be the required point.

    The distance of the point fromx-axis is 2 2 .P Py z

    The distance from the point (1, 1, 2) is

  • 8/12/2019 JEE Knocout Test QP_Solution_Dec22 (1)

    22/76

    22 2( 1) 1 ( 2)P P Px y z

    2 2 2 2 2( 1) ( 1) ( 2)P P P P Py z x y z

    2 2 2 4 6 0P P P Px x y z

    Therefore, the locus of pointPis

    2 2 2 4 6 0x x y z

    Correct Option:(3).

    Question 26: If the axes are rotated through 60 in the anticlockwise sense, find the transformed form of

    the equation 2 2 2x y a .

    (1) 2 2 23 3 2X Y XY a (2) 2 2 2X Y a

    (3) 2 2 22 3 2Y X XY a (4) 2 2 22 3 2X Y XY a

    Chapter: Rectangular Coordinate System

    Level of Difficulty: Moderate

    Solution: Let ( , )x y and ( , )X Y be the old and the new coordinates, respectively. Since, the axes are

    rotated in the anticlockwise direction, 60 . Therefore,

    cos60 sin60

    sin60 cos60

    x X

    y Y

    1 32 2

    3 1

    2 2

    x X

    y Y

    3

    2 2

    3

    2 2

    XY

    x

    y YX

    3 3and

    2 2 2 2

    x Yx Y y X

    2 2

    23 3

    2 2 2 2

    X yY X a

    2 2 2 2( 3 2 3 ) (3 2 3 )X Y XY X Y XY yc

    2 2 22 2 4 3 4X Y XY a

  • 8/12/2019 JEE Knocout Test QP_Solution_Dec22 (1)

    23/76

    2 2 22 3 2Y X XY a

    which is the required equation

    Correct Option:(3).

    Question 27:The straight lines 3 4 0, 3 4 0x y x y and 0x y form a triangle which is

    (1) equilateral (2) right-angled

    (3) acute-angled and isosceles (4) obtuse-angled and isosceles

    Chapter: Straight Lines and Pair of Lines

    Level of Difficulty: Moderate

    Solution: The following figure depicts the condition. By observation from the figure, ABC is clearly an

    obtuse angled and isosceles triangle.

    Correct Option:(4).

    Alternate Solution: The following figure depicts the condition.

  • 8/12/2019 JEE Knocout Test QP_Solution_Dec22 (1)

    24/76

    From the figure, we get

    A: 3 4 and 2; 2x y y x x y

    B: (1,1) by solving the equations.

    C: 3 4 0 and 2; 2x y y x x y

    Thus,

    2 2

    1 9 10

    4 4 4 2

    10 10 16(2)cos 0

    2( 10)( 10)

    AB BC

    AC

    B

    Therefore, the given triangle is isosceles and obtuse angled triangle.

  • 8/12/2019 JEE Knocout Test QP_Solution_Dec22 (1)

    25/76

    Question 28: Find the equation of the circle whose diameter is the common chord of the circles

    2 2 2( )x a y a and 2 2 2( )x y b b

    (1) 2 2 2 2( )( ) 2 ( ) 0x y a b ab bx ay (2) 2 2 2 2( )( ) 2 ( ) 0x y a b ab bx ay

    (3)2 2 2 2

    ( )( ) 2 ( ) 0x y a b ab bx ay (4) 2 2 2 2

    ( )( ) 2 ( ) 0x y a b ab ax by

    Chapter: Circle

    Level of Difficulty: Easy

    Solution: Since 2 2 2( )x a y a , it is concluded that this circle has centre on x-axis and the circle passes

    through origin.

    Since 2 2 2( )x y b b , it is concluded that this circle has centre on y-axis and the circle passes through

    origin.

    Therefore,

    2 2 2 0 (1)x y ax

    2 2 2 0 (2)x y by

    Subtracting Eq. (2) from Eq. (1), we get the common chord of the two circles:

    (3)ax by

    Solving Eqs. (1) and (3); we get the point of intersection of the two given circles:

    22 2 0

    ax x ax

    b

    2

    20 or 2 0

    ax x x a

    b

    2

    2 2

    20or

    abx x

    a b

    2

    2 2

    20or

    a by y

    a b

    Therefore, the equation of circle having (0, 0) and2 2

    2 2 2 2

    2 2,

    ab a b

    a b a b

    as end points of one of its diameter

    is

    2 2

    2 2 2 2

    2 2( 0) ( 0) 0

    ab a bx x y y

    a b a b

    2 22 2

    2 2 2 2

    2 20

    ab a bx x y y

    a b a b

  • 8/12/2019 JEE Knocout Test QP_Solution_Dec22 (1)

    26/76

    2 2 2 2( )( ) 2 ( ) 0x y a b ab bx ay

    Correct Option:(1).

    Question 29:Tangents are drawn from the point (4, 2) to the curvex2

    + 9y2

    = 9. The angle between these

    tangents is

    (1)3 3

    5 17 (2)

    43

    10

    (3)43

    5 (4)

    3

    17

    Chapter: Ellipse

    Level of Difficulty: Hard

    Solution

    The combined equation of pair of tangents drawn from a point (x1,y1) to the ellipse2 2

    2 2 1 0

    x yS

    a b is

    S12= SS11. Therefore,

    2 2 22 21 1 1 1

    2 2 2 2 2 2

    2 2 22 2

    2 2

    2

    1 1 1

    4 42 1 1 2 1

    9 9 9

    3 7 16 8 36 52 0

    2tan

    xx yy x yx y

    a b a b a b

    x xy y

    x y xy x y

    h ab

    a b

    where, a= 3, b= 7 and h=8. Therefore,

    2 64 21 43tan

    10 5

    Note:is acute angle between the pair of tangents. Therefore,

    2 2 2 2( ) 2 2 2a b c a b c ab ac bc

    Correct Option:(2).

    Quick Tip

    S12= SS11

    22

    tan h ab

    a b

    , that is,

  • 8/12/2019 JEE Knocout Test QP_Solution_Dec22 (1)

    27/76

    1 2

    1 2

    21 2 1 2

    1 2

    tan1

    ( ) 4

    1

    m m

    m m

    m m m m

    m m

    Alternate Solution: Any line passing through the point (4, 2) is given by

    2 ( 4)y m x

    4 2y mx m

    For this line to be tangent to the given ellipse, put this yinto the equation of the ellipse and make

    D= 0

    That is,

    2

    2

    2 2 2 2

    ( 4 2) 19

    (1 9 ) (36 72 ) 16(9) 16(9) 27 0

    x mx m

    m x x m m m m

    Now,

    2

    2 2 2 2

    2 2 2 2

    2 2 2 4 2

    2

    0 4 0

    (36 72 ) 4(1 9 )(16.9 16.9 27) 0

    (36 ) (1 2 ) 36(1 9 )(16 16 3) 0

    (1 4 4 ) 36(16 16 3 9.16 9.16 27 ) 0

    7 16 3 0

    D B AC

    m m m m m

    m m m m m

    m m m m m m m m

    m m

    Now,

    21 2 1 21 2

    1 2 1 2

    2

    2

    ( ) 4tan

    1 1

    16 34.

    7 2tan

    31

    7

    7 16 4.3.7

    10 7

    14(43)

    10

    43

    5

    m m m mm m

    m m m m

    where is the acute angle between the tangents.

  • 8/12/2019 JEE Knocout Test QP_Solution_Dec22 (1)

    28/76

    Question 30:The chord of contact of a point A AA( , )x y of2 4y x passes through (3, 1) and point A lies

    on 2 2 25x y . Then

    (1)2

    A A5 24 11 0x x (2)2

    A A13 8 21 0x x

    (3) 2A A5 24 61 0x x (4)2A A13 21 31 0x x

    Chapter: Parabola

    Level of Difficulty: Easy

    Solution: The following figure depicts the condition. Chord of contact of a point A AA( , )x y with respect

    to 2 4y x is A A2( ).y y x x Since this chord passes through the point (3, 1), we have

    A A2( 3)y x

    AB and AC are tangents to the parabola.

    BC is chord of contact of point A with respect to the parabola 2 4 .y ax

    Given that point A lies on 2 2 25x y , we have

  • 8/12/2019 JEE Knocout Test QP_Solution_Dec22 (1)

    29/76

    2 2A A

    2 2A A

    2 2A A A

    2A A

    2A A

    25

    4( 3) 25

    4( 9 6 ) 25

    5 24 36 25 0

    5 24 11 0

    x y

    x x

    x x x

    x x

    x x

    Quick Tip: The chord of contact of point A AA( , )x y with respect to2 4 0y ax is

    A A2( )( ) 0y y a x x

    Correct Option:(1).

  • 8/12/2019 JEE Knocout Test QP_Solution_Dec22 (1)

    30/76

    Part B Physics

    Question1. The figure below shows two different arrangements of materials 1, 2 and 3 to form a wall. All

    the three slabs are of equal thickness. The thermal conductivities are 1 2 3k k k . The left side of the wall

    is 20 o C higher than the right side. Which of the following statements is correct for steady state heat flow

    across the wall?

    (1) The temperature difference across 1 will be the greatest in both the arrangements.

    (2) The rate of heat transfer across the wall will be the same for both the arrangements.

    (3) The temperature difference across 1 will be greater in the first arrangement than in the second

    arrangement.

    (4) The rate of heat transfer across the wall will be greater in the first arrangement than in the second

    arrangement.

    Chapter:Conduction

    Level of Difficulty:Moderate

    Solution

    In both the arrangements, the three materials are in series. Since, the temperature difference across the

    wall is the same for both the arrangements, the heat current will also be the same for both. Hence, the rate

    of heat transfer across the wall will be the same for both the arrangements.

    Correct Option:(2)

    Question2.A bubble from the tank of a scuba diver contains43.5 10 mol of gas. The bubble expands

    as it rises to the surface from a freshwater depth of 10.3 m. Assuming that the gas is an ideal gas and the

    temperature remains constant at 291 K, the amount of heat that flows into the bubble is

    (1)1

    0.423 ln2

    (2)1

    0.846 ln2

  • 8/12/2019 JEE Knocout Test QP_Solution_Dec22 (1)

    31/76

    (3) 0.846 ln(2) (4) 0.423 ln(2)

    Chapter:Thermodynamics

    Level of Difficulty:Hard

    Solution

    The process of expansion of the gas is an isothermal process. Therefore, 0U andQ W .

    2

    1

    ln V

    Q W nRT V

    Since the process is isothermal,

    2 1

    1 2

    V P

    V P ,

    where1Pis the pressure inside the bubble at the bottom and 2P is the pressure at the top of the surface.

    2 1 atmP .

    1 2 Pressure of 10.3 m columnP P of water

    = 1atm + 1 atm [since pressure of 10.3 m column of water = 1

    atm]

    2 atm

    Hence,

    2 1

    1 2

    2V P

    V P

    So, the amount of heat that flows into the bubble will be

    2

    1

    4

    ln

    3.5 10 8.31 291 ln 2

    0.846 ln 2

    Q W

    VnRT

    V

    Correct Option:(3)

    Question3. An ideal gas is taken along the cycle ABCA as shown in the figure given below with

    1 40PaP , 3

    1 1.0mV , 2 10PaP , and3

    2 4.0mV . The amount of heat transferred into the system

    during the entire cyclic process is

  • 8/12/2019 JEE Knocout Test QP_Solution_Dec22 (1)

    32/76

    (1)45 J (2) 45 J

    (3) 55 J (4) 55 J

    Chapter:Thermodynamics

    Level of Difficulty:Easy

    Solution

    For the cyclic process, change in internal energy 0U . So, using first law of thermodynamics we get

    Q W

    Now, the work done will be

    W = the area enclosed inside the cycle

    1 2 2 1

    3 3

    1( )( )

    21

    (40Pa 10Pa)(4.0m 1.0m )2

    45 J

    P P V V

    Thus, heat is transferred into the system during the entire cyclic process will be 45J.Q

    Correct Option:(2)

    Question4.A ball suspended by a light inextensible string from a ceiling is shifted by an angle 0 (as

    shown in the figure below) so as to strike the vertical wall normally with the coefficient of restitution e.The maximum angle made by the string with vertical after the first collision is

    0

    O

  • 8/12/2019 JEE Knocout Test QP_Solution_Dec22 (1)

    33/76

    (1) 1 2 0cos 1 (1 cos )e (2) 1 2 0sin 1 (1 sin )e

    (3) 0 (4) 1 2 0cos (1 cos )e

    Chapter:Collisions

    Level of Difficulty:Hard

    Solution

    0

    O

    h1

    l

    Let the mass of ball be mand length of the string suspending the ball be l. Then the height h1with respect

    to the position at collision is given by

    01 1 cosh l

    Let v1be the speed of the ball before the first collision. Conserving mechanical energy up to first collision

    i i f f K U K U

    21 11

    0 02

    mgh mv

    02

    1

    11 cos

    2gl v

    01 2 1 cosv gl

    If v2is the speed of the ball just after the collision, then

    2 1 02 1 cosv ev e gl

    If ball rises to height h2after the collision then the angle made by string with the vertical is related to it

    as

    2 1 cosh l

    Conserving mechanical energy after the first collision up to rise to height h2

    i i f f K U K U

    22 21

    0 02

    mv mgh

    021 2 1 cos 1 cos

    2e gl gl

  • 8/12/2019 JEE Knocout Test QP_Solution_Dec22 (1)

    34/76

    02 1 cos 1 cose

    2 0cos 1 1 cose

    1 02cos 1 1 cose

    Correct Option:(1)

    Quick Tip: By substituting e= 1, you should get 0and by putting e= 0, you should get 0. That you get

    only with 1 2 0cos 1 (1 cos )e and 1 2 0sin 1 (1 sin )e and no other option. By realizing the

    relation of height with angular deviation, that is, 1 cosh l , you can zero in the right option of

    1 2 0cos 1 (1 cos )e .

    Question5.Planet A has mass

    24

    3.00 10 kgM and radius 7

    2.00 10 mR , and it completes a fullrotation in time 35.0hT . What is the free-fall accelerationgon its equator? (Take G=

    11 2 26.67 10 Nm / kg )

    (1) 20.500ms (2) 20.050ms

    (3) 20.450ms (4) 20.550ms

    Chapter:Gravitation

    Level of Difficulty:Moderate

    Solution

    The gravitational acceleration on the planets surface is given by

    g 2

    11 24

    7 2

    2

    (6.67 10 )(3.00 10 )

    (2.00 10 )

    0.500ms

    GMa

    R

    The centripetal acceleration of any mass on the planets surface at the equator is

    2

    c

    2

    2 7

    2

    2

    2

    4 (2.00 10 )

    (35 3600)

    0.0497 ms

    a R

    RT

  • 8/12/2019 JEE Knocout Test QP_Solution_Dec22 (1)

    35/76

    In rotational frame any mass will experience a centrifugal force (= mac) which is directed away from the

    center. Hence, achas to be subtracted from ag. Free-fall acceleration at the equator of the planet will be

    g c

    2 2

    2

    0.500ms 0.0497ms

    0.450ms

    g a a

    Correct Option:(3)

    Question6.The position of a particle moving along thex-axis is given by 37.8 9.2 2.1x t t , with x in

    meters and tin seconds. What is the velocity at 3st ?

    (1) 121 ms (2) 135 ms

    (3) 147.5 ms (4) 154ms

    Chapter:Motion along a Straight LineLevel of Difficulty:Easy

    Solution

    Position of a particle moving along thex-axis is

    37.8 9.2 2.1x t t

    The velocity of the particle will be

    3

    2

    7.8 9.2 2.1

    9.2 6.3

    dxv

    dt

    d

    t tdt

    t

    At 3st , the velocity of the particle is

    2 19.2 6.3(3 ) 47.5 msv

    Correct Option:(3)

    Question7.A particle is launched into projectile motion over level ground. At point A, 3.0m above

    ground level, the particle has velocity 1 1

    (5.0 ms ) ( 6.0 ms )v i j

    . What was its angle of launch?

    (1) 1 6.25

    tan5

    (2) 1 3.65

    tan5

    (3) 1 8.59

    tan5

    (4) 1 9.74

    tan5

    Chapter:Motion in Two Dimensions and Projectile Motion

  • 8/12/2019 JEE Knocout Test QP_Solution_Dec22 (1)

    36/76

    Level of Difficulty: Moderate

    Solution

    At h=3.0m above ground level,

    1 1 (5.0 ms ) ( 6.0 ms )v i j

    Let the initial velocity be x yu u i u j .

    Then 15.0 msxu because for the projectile, the horizontal orx-component of velocity remains constant.

    Using third kinematics relation along the vertical direction, we get

    2 2

    1

    ( 6) 2(9.8) (3.0)

    9.74 ms

    y

    y

    u

    u

    The required angle of launch will be

    1 1 9.74tan tan 5

    y

    x

    u

    u

    Correct Option:(4)

    Question8.A flat body of mass 2.00 kg is sliding on a frictionless surface, as two constant forces act on

    it: 1(10N)F j and 2 ( 4N)F j . At 0t , the velocity of the body is

    1

    0(4.00 ms )v i . What is the

    angle of the bodys velocity (relative to the positivex-direction) at 3.00st ?

    (1) 1 3

    tan2

    (2) 1

    3tan

    4

    (3) 1 9

    tan4

    (4) 1 5

    tan8

    Chapter:Newtons Laws of Motion

    Level of Difficulty:Moderate

    Solution

    The acceleration of the body will be

    1 2

    2

    10 4

    2.00

    (3.00 )ms

    F Fa

    m

    j j

    j

    So, to find out the velocity at 3.00st , we can write

  • 8/12/2019 JEE Knocout Test QP_Solution_Dec22 (1)

    37/76

    0

    4.00 (3.00 )(3.00)

    4.00 9.00

    v v a t

    i j

    i j

    Thus the components of velocity are14.00 ms

    xv and 19.00 ms .yv

    If be the angle made by this velocity with the positivex-direction, then

    1

    tan

    9.00

    4.00

    9tan

    4

    y

    x

    v

    v

    Correct Option:(3)

    Question9. A body is rotating around a fixed axis with angular velocity 3 rad s1with constant angular

    acceleration of 1 rad s2at some time. Find the magnitude of acceleration of a particle 5 m away from the

    axis after the body has turned by 90.

    (1) 2 25 1 9 ms (2) 25 9 ms

    (3)25 ms (4) 25 82 ms

    Chapter:Rotational KinematicsLevel of Difficulty:Moderate

    Solution

    Initial angular velocity 0 3 rad s1.

    Angular acceleration= 1 rad s2.

    If angular velocity after turning by = 90 is , then we have,

    2 2

    0 2

    2

    2

    9 2 1 2

    9

    Acceleration will have two components, tangential and centripetal. The centripetal component of

    acceleration is

    2c29 5 msra

    and the tangential component is

  • 8/12/2019 JEE Knocout Test QP_Solution_Dec22 (1)

    38/76

    2

    t 5 1 msa r

    The net acceleration will be

    2 2

    c t

    2 2

    2 2

    5 9 1

    5 1 9 ms

    a a a

    Correct Option:(1)

    Question10. Two boxes, A and B, are connected to each other as shown in the figure given below.

    The system is released from rest and the box B falls through a distance of1.00m . The surface of the table

    is frictionless. What is the kinetic energy of box B just before it reaches the floor?

    (1) 2.45J (2) 4.90J

    (3) 9.80J (4) 29.4J

    Chapter:Work and Energy

    Level of Difficulty:Moderate

    Solution

    Let Tbe the tension in the string as shown in the figure given below.

  • 8/12/2019 JEE Knocout Test QP_Solution_Dec22 (1)

    39/76

    We can write the force equation for both the masses as:

    B B

    A

    (1)

    (2)

    m g T m a

    T m a

    Adding Eq. (1) and Eq. (2), we get

    B B A

    B

    A B( )

    m g m a m a

    ma g

    m m

    Substituting value of ain Eq. (2), we get

    A B

    A B( )

    3.00 1.00

    3.00 1.00

    3.00

    4.00

    m mT g

    m m

    g

    g

    Since the system is released from rest, initial kinetic energy i 0J.K

    Final kinetic energy of box B is equal to the net work done on it by the tension force and the gravitational

    force.

    f

    B

    B

    2

    ( ) (1.00)

    3.001.00 (1.00) [where 9.8ms ]

    4.00

    9.8

    4

    2.45 J

    K W

    Fd

    m ad

    m g T

    g g g

    Correct Option:(1)

    Question11. Two uniform steel bars are freely hanging from a ceiling. The length of the second bar is

    double that of the first. The ratio of elongation of first bar to that of second is(1) 1 : 4 (2) 4 : 1

    (3) 2 : 1 (4) 1 : 2

    Chapter:Elasticity

    Level of Difficulty:Moderate

    Solution

  • 8/12/2019 JEE Knocout Test QP_Solution_Dec22 (1)

    40/76

    Let the first bar be of massM, lengthL, cross sectionA, mass per unit length mand Youngs modulus Y.

    Therefore,

    M =L

    Elongation of segment dx, at a distancexfrom the free bottom is given by

    dxdx Tx

    AY

    dxmxg

    AY

    Mx dxg

    L AY

    Lx dxg

    L AY

    dxxg

    AY

    Elongation of a freely hanging body

    0

    0

    2

    0

    2

    2

    2

    L

    L

    L

    x dx

    dxxg

    AY

    gxdx

    AY

    g x

    AY

    gL

    AY

    Therefore,

    2x L

    Hence, if length is doubled, elongation will quadruple.

    Correct Option:(1)

    Question12.The height of liquid in a capillary tube depends upon

    (1) the outer radius of the tube. (2) the inner radius of the tube.

    (3) the temperature of the tube. (4) the length of the tube.

    Chapter:Surface Tension and Viscosity

    Level of Difficulty:Easy

    Solution

    Rise of liquid in a capillary tube is given by the relation

  • 8/12/2019 JEE Knocout Test QP_Solution_Dec22 (1)

    41/76

    2 cosTh

    r g

    where r= inner radius of tube, T= surface tension and= density of liquid. Thus, height of liquid

    depends on the inner radius of the capillary tube not the outer radius.

    Correct Option:(2)

    Question13.A particle executes SHM alongx-axis about the center atx=awith frequencyf. Initially

    the particle is at rest at origin. Its equation of motion is

    (1) (cos 2 1)x a ft (2) (1 cos 2 )x a ft

    (3) 2 (cos 2 1)x a ft (4) 2 (1 cos 2 )x a ft

    Chapter:Simple Harmonic Motion

    Level of Difficulty:Moderate

    Solution

    Aboutx = 0, SHM equation is

    sin 2 fx a t

    For particle initially at rest at maximum positive displacement,2

    .

    sin 2 cos 22

    x a t a tf f

    (1)

    Shifting axis of SHM tox = a, Eq. (1) would be

    cos 2

    cos(2 )

    os2 1c

    x

    fx a t

    tf

    a a t

    a

    f

    a

    Correct Option:(1)

    Question14. A spherical source of radius 1 m emits sound equally in all directions in a non-dispersive

    medium. The ratio of the magnitude of the amplitudes of the waves at two points at a distance of 25 m

    and 36 m from the source is

    (1) 36:25 (2) 5:6

    (3) 6:5 (4) 216:125

    Chapter:Sound Waves

    Level of Difficulty:Moderate

  • 8/12/2019 JEE Knocout Test QP_Solution_Dec22 (1)

    42/76

    Solution

    Power transfer across a spherical surface with the same center as the source would be same across all such

    surfaces by conservation of energy. Intensity at any point at a distance of rfrom the center, if power

    generated by the source isP, would be

    24

    PI

    r

    Since 2I A when other independent parameters remain same, so we can write

    2

    2

    1A

    r

    1A

    r

    Hence, ratio would be 36:25 because amplitude is inversely proportional to distance.

    Correct Option:(1)

    Question15.Two 3.25 Fcapacitors are connected in series across the terminals of an 50.0Hz

    alternating current (AC) generator that has a peak voltage of 244 V . What is the rms current in the

    circuit?

    (1) 0.124A (2) 0.088A

    (3) 0.308A (4) 0.248A

    Chapter:Alternating Current

    Level of Difficulty:Moderate

    Solution

    Since the capacitors are in series, the effective capacitance of the circuit will be

    (3.25)(3.25)1.625F

    (3.25 3.25)C

    Hence, capacitive reactance will be

    6

    3

    1

    21

    2 (50.0)(1.625 10 )

    1.96 10

    CX

    fC

    Peak current will be,

  • 8/12/2019 JEE Knocout Test QP_Solution_Dec22 (1)

    43/76

    00

    3

    244

    1.96 10

    0.124 A

    C

    VI

    X

    Therefore, the rms current will be

    0rms

    2

    0.124

    2

    0.088 A

    II

    Correct Option:(2)

    Question16.A stream of electrons is going round a circle in the counter clockwise direction as shown in

    the figure below.

    At the center of the circle, they produce a magnetic field that is directed

    (1) into the page. (2) out of the page.

    (3) to the left. (4) to the right.

    Chapter:Magnetic Field due to a Current

    Level of Difficulty:Easy

    Solution

    The direction of the conventional current is opposite to the direction of the flow of the electrons. So,applying the right hand rule, we conclude that the magnetic field at the center will be directed into the

    page.

    Correct Option:(1)

  • 8/12/2019 JEE Knocout Test QP_Solution_Dec22 (1)

    44/76

    Question17.In Youngs double slit experiment, blue light ( 440 nm) gives a second order bright

    fringe at a certain location on a flat screen. What wavelength of visible light would produce a dark fringe

    at the same location? Assume that the range of visible wavelengths extends from 380 nm to 750 nm .

    (1) 467 nm (2) 587 nm

    (3) 671 nm (4) 736 nm

    Chapter:Light Waves

    Level of Difficulty:Moderate

    Solution

    Since it is a second-order bright fringe, we have

    2

    2 2(440) 880D D Dy

    d d d

    where dis separation between the slits andDis distance of screen from slits.

    For getting a dark fringe at the same position,

    ( 1 / 2) 880

    1880 (1)

    2

    m D D

    d d

    m

    where 0, 1, 2,...m

    For 0m , using Eq. (1) we get

    10 880

    2

    1760 nm

    For 1m , using Eq. (1) we get

    11 880

    2

    587 nm

    ,

    and for 2m , using Eq. (1) we get

    12 880

    2352 nm

    So, the only value that falls within the given range of 380 nm to 750 nm is 587 nm.

    Correct Option:(2)

  • 8/12/2019 JEE Knocout Test QP_Solution_Dec22 (1)

    45/76

    Question18.A diverging lens of focal length 14.0 cm is 30.0 cm to the left of a converging lens of focal

    length12.0 cm . An object is placed 16.0 cm to the left of the diverging lens. How far is the final image

    from the converging lens (after refraction through the converging lens)?

    (1) 7.5 cm (2) 15.7 cm

    (3) 17.6 cm (4) 27.4 cm

    Chapter:Refraction

    Level of Difficulty:Hard

    Solution

    Applying lens formula for the diverging lens,

    1 1 1

    1 1 1

    1 1

    16.0 14.0

    15

    112

    7.47 cm

    v u f

    v u f

    v

    This image becomes the virtual object for the second (converging) lens. Thus, the object distance for the

    converging lens = 7.47 30.0 37.47 cm.

    We can now apply the lens formula to the converging lens.

  • 8/12/2019 JEE Knocout Test QP_Solution_Dec22 (1)

    46/76

    1 1 1

    1 1 1

    1 1

    37.47 12.0

    37.47 12.0

    25.47

    17.6 cm

    v u f

    v u f

    v

    Thus, the final image will be formed 17.6 cm to the right of the converging lens.

    Correct Option:(3)

    Question19. If(n+ 1) divisions on the vernier scale (VSD) of a vernier calipers coincide with ndivisions

    on the main scale (MSD) and each division on the main scale is of cunits, then the least count of

    instrument is

    (1)1

    c

    n (2)

    2

    c

    n

    (3)c

    n (4)

    1

    c

    n

    Chapter: Experimental Skills

    Level of Difficulty: Moderate

    SolutionLeast count (LC) = 1 MSD1 VSD

    In the given question

    (n) MSD (n+ 1) VSD

    1VSD MSD1

    n

    n

    So, the least count will be

    LC = I MSD MSD1

    n

    n

    1MSD

    1

    1MSD

    1

    n n

    n

    n

    Since each division on the main scale is of cunits

  • 8/12/2019 JEE Knocout Test QP_Solution_Dec22 (1)

    47/76

    LC1

    c

    n

    Correct Option:(1)

    Question20.A wire of cross-sectional radius ais bent to form a circle of radiusR(R>> a). A charge Qis

    uniformly distributed on the ring. An infinite line of positive charges of linear charge density passes

    through center of the ring, perpendicular to the plane of the ring. If Youngs modulus of the material of

    the wire is Y, then the change in length of the wire after placing the infinite line of charge at the center of

    charged ring is

    (1)2 2

    02

    Q

    a Y

    (2)

    2 2

    0

    Q

    a Y

    (3)2 2

    0

    2 Q

    a Y

    (4)

    2 2

    0

    3

    2

    Q

    a Y

    Chapter:Electric Field

    Level of Difficulty:Hard

    Solution

    Consider an infinitesimal element of the circle, as shown in the figure below.

    Charge on the infinitesimal element =2

    Qdx

    R

    Electric field due to infinite line of charges at distanceRis given by

    02E

    R

    0

    2 sin2 2

    QT dx

    R R

    0

    (2 )2 2

    QT dx

    R R

  • 8/12/2019 JEE Knocout Test QP_Solution_Dec22 (1)

    48/76

    Since is small

    02 2

    dx QT dx

    R R R

    204

    QT

    R

    2/

    /

    T aY

    L L

    2

    2T RL

    a Y

    2 2

    02

    QL

    a Y

    Correct Option:(1)

    Question21. Read the two given statements S1 and S2 carefully and mark the correct options

    S1:The potential difference between the shells of two concentric charged spherical shells depends on

    charge of inner shell.

    S2:Potential due to charge of outer shell remains same at every point inside the sphere.

    (1) S1 and S2 are True and S2 is the correct explanation for S1.

    (2) S1 is True and S2 is True but S2 is NOT a correct explanation for S1.

    (3) S1 is True and S2 is False.

    (4) S1 is False and S2 is True.

    Chapter:Electric Potential

    Level of Difficulty:Easy

    Solution

    Consider the two concentric shells A and B shown in the figure below.

    Potential due to shell A

  • 8/12/2019 JEE Knocout Test QP_Solution_Dec22 (1)

    49/76

    1 2A

    0 2

    1

    4

    Q QV

    R

    Potential due to shell B

    1 2B

    0 1 2

    1

    4

    Q QV R R

    The potential difference will be

    B A 1

    0 1 2

    1 1 1

    4V V V Q

    R R

    Hencethe potential difference between the shells of two concentric charged spherical shells depends on

    charge of inner shell because potential due to charge of outer shell remains the same at every point inside

    the sphere.

    Correct Option:(1)

    Question22.A conducting wire of length 1 m, being used as a potentiometer wire, has radius linearly

    changing from r to 2racross its ends. The distance of the null point from the end with radius r,if the

    resistances in the gaps are in the ratio 2 : 3, is

    (1) 25 cm (2) 40 cm

    (3) 20 cm (4) 33.33 cm

    Chapter:Electric Current in Conductors

    Level of Difficulty: Hard

    Solution

    Since the radius of the wire increases linearly with distancexfrom left end, we have the radiusRat a

    distancexas

    R= r+xr

    l

    The resistance of an element dxat a distancexis

    22dx dxrR xr

    rl

  • 8/12/2019 JEE Knocout Test QP_Solution_Dec22 (1)

    50/76

    If the null point occurs at a distance x from the left end, we have the ratio of the two parts (left and

    right) in the ratio 2 : 3 . This implies

    2

    0

    2

    23

    x

    x

    dx

    xrr

    ldx

    xrr

    l

    32

    0( )

    xdx

    l x

    2 2( )l

    x

    dx

    l x

    3

    01 1

    2

    x

    x ll x l x

    3 1 1 1 122l l x l x l

    Substituting l= 100 cm and simplifying above expression, we get

    1 1 1 13 2

    100 100 100 200x x

    x= 25 cm

    Correct Option:(1)

    Question23.When a shunt of resistance 2is connected in parallel to galvanometer, its range to measure

    current becomes 11 times. Then the resistance required to be connected in series to convert it into

    voltmeter of range 20 times is

    (1) 380 (2) 280

    (3) 180 (4) 480

    Chapter:Electric Current in Conductors

    Level of Difficulty: Easy

    Solution

    Now the shunt Scan be expressed as

    g

    g

    1

    RS

    I

    I

  • 8/12/2019 JEE Knocout Test QP_Solution_Dec22 (1)

    51/76

    g2

    11 1

    R

    g 20R

    The required resistance can be calculated using

    g

    g

    1V

    R RV

    20 (20 1)

    380

    Correct Option:(1)

    Question24.The graph given below, plots photocurrent versus anode potential for a cathode with 4eVwork function. The energy of the incident photon is

    (1) 6 eV (2) 4 eV

    (3) 2 eV (4) 8 eV

    Chapter:Photoelectric effect

    Level of Difficulty:Easy

    Solution

    Given = 4eV

    From the graph, stopping potential V0= 2V. This implies that

    Kmax= eV0= 2 eV

    But from Einsteins photoelectric equation

    h= Kmax+

    h= (4+ 2)eV= 6eV

    Correct Option:(1)

  • 8/12/2019 JEE Knocout Test QP_Solution_Dec22 (1)

    52/76

    Question25.An inductanceLand a resistanceRare connected in series to an ideal battery. A switch in

    the circuit is closed at time 0, at which time the current is zero. The rate of increase of the energy stored in

    the inductor is maximum

    (1)just after the switch is closed.

    (2) at time /t L R after the switch is closed.

    (3) at time 2 /t L R after the switch is closed.

    (4) at time ( / ) ln 2t L R after the switch is closed.

    Chapter:Inductor

    Level of Difficulty:Hard

    Solution

    The transient current in anLRcircuit is given by

    1 exp /

    V t

    I R L R

    Therefore,

    exp/

    dI V t

    dt L L R

    Energy stored in the inductor is given by

    21

    2E LI

    Rate of change of energy,

    2

    exp 1 exp/ /

    dE di V t t L I

    dt dt R L R L R

    2 2

    2 exp 1 2exp

    / /

    d E V t t

    dt L L R L R

    Now, fordE

    dtto be maximum,

    2

    2 0

    d E

    dt

    Thus,

    2

    exp 1 2exp 0/ /

    V t t

    L L R L R

    1exp

    / 2

    ln 2/

    ( / ) ln 2

    t

    L R

    t

    L R

    t L R

  • 8/12/2019 JEE Knocout Test QP_Solution_Dec22 (1)

    53/76

    Correct Option:(4)

    Question26.For a logic gate, the voltage waveforms A and B are shown as input in the figure given

    below and C as output. The logic circuit gate is

    (1) OR gate

    (2) AND gate

    (3)NAND gate(4)NOR gate

    Chapter:Digital Electronics

    Level of Difficulty:Moderate

    Solution

    The truth table corresponding to waveforms is as shown here. It indicates that the given logic circuit gate

    is AND gate.

    A B C

    0 0 0

    1 0 0

    0 1 0

    1 1 1

  • 8/12/2019 JEE Knocout Test QP_Solution_Dec22 (1)

    54/76

    Hence, the correct option is (2).

    Correct Option:(2)

    Question27.The atomic mass of 10B is 10.811 u. The binding energy of 10B nucleus is (given that the

    mass of electron is 0.0005498 u, the mass of proton is mp= 1.007276 u, and the mass of neutron is mn=

    1.008665 u)

    (1)681.201 MeV (2) 678.932 MeV

    (3) 378.932 MeV (4) None of these

    Chapter:Atomic Nucleus

    Level of Difficulty:Moderate

    Solution

    Mass of 10B nucleus isM= 10.811u

    Mass of proton mp= 1.007276 u

    Mass of neutron mn= 1.008665u

    The number of protons and neutrons in10

    B nucleus is 5 each.

    The mass defect will be

    1 007276 5 1 008665 10 811

    5 03638 5 043325 10 811

    10 079705 10 811

    0 731295u

    5 5

    5

    p nm

    . . .

    . . .

    .

    m

    .

    m

    .

    M

    The binding energy will be

    2

    931.5 MeV

    0.731295 931.5 MeV

    681.201MeV

    E mc

    m

    Correct Option:(1)

  • 8/12/2019 JEE Knocout Test QP_Solution_Dec22 (1)

    55/76

    Question28.In case ofp-njunction diode at high value of reverse bias, the current rises sharply. The

    value of reverse bias is known as

    (1) cut-off voltage.

    (2) Zener voltage.

    (3) inverse voltage.

    (4) critical voltage.

  • 8/12/2019 JEE Knocout Test QP_Solution_Dec22 (1)

    56/76

    Chapter:Semiconductors

    Level of Difficulty:Easy

    Solution

    In case ofp-njunction diode at high value of reverse bias, the current rises sharply. The value of reverse

    bias is known as Zener voltage.

    The Zener diode or breakdown diode, as it is sometimes called, is basically the same as the

    standard p-n junction diode but is specially designed to have a low predetermined reverse breakdown

    voltage that takes advantage of this high reverse voltage. TheZener diodeis the simplest types of voltage

    regulator and the point at which a Zener diode breaks down or conducts is called the Zener voltage.

    Correct Option:(2)

    Question29.A T.V. transmission tower has a height of 180 m. The broadcast is available up to a distance

    of (Radius of Earth 6.4 106m)

    (1) 48 km (2) 24 km

    (3) 34 km (4) 68 km

    Chapter:Communication Systems

    Level of Difficulty:Easy

    Solution

    The range of broadcast can be calculated using relation

    6

    3

    2

    2 180 6.4 10

    48 10 m

    48km

    d hR

    Correct Option:(1)

    Question30.Read the two statements S1 and S2 are given below and answer the correct option

    S1: A normal human eye can clearly see all the objects just beyond a certain minimum distance.

    S2: The human eye has the capacity to adjust suitably the focal length of its lens to a certain extent.

    (1) S1 is True, S2 is True and S2 is a correct explanation for S1.

    (2) S1 is True, S2 is True but S2 is NOT a correct explanation for S1.

    (3) S1 is True but S2 is False.

    (4) S1 is False but S2 is True.

    Chapter:Optical Instruments

  • 8/12/2019 JEE Knocout Test QP_Solution_Dec22 (1)

    57/76

    Level of Difficulty:Moderate

    Solution

    A normal human eye can clearly see all the objects just beyond a certain minimum distance which is 25

    cm for normal eye and it is known as near point. Hence statement 1 is correct.

    Accommodation is the adjustment to the thickness of the lens in the eye to ensure that the image on the

    retina is sharp. When thickness of the lens changes the focal length of the lens also changes. Thus, the

    human eye has the capacity to adjust suitably the focal length of its lens to a certain extent. Therefore

    statement 2 is also correct.

    Statement 1 and statement 2 are two independent statements. Therefore, statement 2 is not a correct

    explanation of statement 1.

    Correct Option:(2)

  • 8/12/2019 JEE Knocout Test QP_Solution_Dec22 (1)

    58/76

    Part CChemistry

    Question 1:Which of the following reactions does not contribute to air pollution in the troposphere?

    (1)4 10 2 2 2C H (g) + 6O (g) CO(g) + 2CO (g) + 5H O(l)

    (2)Lightning

    2 2

    2 2

    N (g) O (g) 2NO

    2NO(g) + O (g) 2NO (g)

    (3)4 22HCHO CH (g) + CO (g)

    (4)3 2Cl + O (g) ClO + O (g)

    Chapter:Environmental Chemistry

    Level of Difficulty:Moderate

    Solution

    The first reaction takes place during incomplete combustion of fossil fuels and releases oxides of carbon

    into the troposphere. The second set of reactions represents formation of oxides of nitrogen in the

    troposphere. The third reaction represents formation of hydrocarbons due to incomplete combustion of

    fuels or anaerobic decomposition of organic matter. The fourth reaction between chlorine radical and

    ozone takes place in the stratosphere and leads to depletion of ozone in the stratosphere. Chlorine free

    radical is obtained from chlorofluorocarbons released into the atmosphere.

    2 2 2F CCl (g) F C Cl(g) + Cl(g)h

    Correct Option: (4)

    Question 2:The four elements Ca, Mg, P and Cl are to be arranged in the order of their increasing

    atomic sizes. Which of the following orders is correct?

    (1) Cl < P < Mg < Ca (2) P < Cl < Ca < Mg

    (3) Ca < Mg < Cl < P (4) Mg < Ca < P < Cl

    Chapter:Classification of Elements and Periodicity of Properties

    Level of Difficulty: Moderate

    Solution

  • 8/12/2019 JEE Knocout Test QP_Solution_Dec22 (1)

    59/76

    Element Atomic

    number

    Number of

    electrons

    Cl 17 17

    P 15 15

    Mg 12 12

    Ca 20 20

    As theZeffincreases along a period, the atomic radius decreases, so the order is Mg < P < Cl. Down the

    group, there is increase in the number of shells, hence the radius increases. So, the order is Mg < Ca.

    Correct Option: (4)

    Question 3:During the concentration of sulphide ores by froth floatation process, the separation of

    sphalerite and galena is achieved by which of the following substances used as depressant?

    (1) Potassium xanthate (2) Sodium cyanide

    (3) Copper sulphate (4) Pine oil

    Chapter:General Principles and Processes of Isolation of Elements

    Level of Difficulty: Moderate

    Solution

    Collector, that is, xanthate acts only on solid particles through adsorption. The two ores show different

    reactivity towards NaCN:

    2 2 4(Soluble complex)

    ZnS 4NaCN Na S Na Zn(CN)

    PbS NaCN No reaction

    Correct Option: (2)

    Question 4:Which one of the following is an electron-deficient species?

    (1) B2H6 (2) NH3 (3) 4AlH

    (4) CH4

    Chapter:p-Block Elements

    Level of Difficulty:Easy

    Solution

    After having 3 center2 electron bond also, B2H6is still electron deficient which can be evidenced by the

    following spontaneous reaction:

    2 6 2 6B H 2THF B H THF

  • 8/12/2019 JEE Knocout Test QP_Solution_Dec22 (1)

    60/76

    The octet/duplet is complete for the central atom in the rest of the hydrides, so they are not electron

    deficient.

    B2H6: NH3:

    4AlH

    : CH4:

    Correct Option: (1)

    Question 5:When spherically symmetrical field of ligands surrounds the central metal ion, then which of

    the following options is correct for the change in energy order of d-orbitals of the central metal ion?

    (1) t2g> eg (2) eg> t2g (3) eg= t2g (4) Cannot be predicted

    Chapter:Coordination Compounds

    Level of Difficulty: Moderate

    Solution

    If a symmetrical field of negative charges surrounds a metal ion, the d-orbitals remain degenerate.

    Correct Option: (3)

    Question 6: Which of the following pair of elements of lanthanoids do not follow the regular trend of

    decrease in radius from Ce to Lu?

    (1) Eu and Gd (2) Sm and Tm (3) Pr and Er (4) Eu and Yb

    Chapter:dandf-Block Elements

    Level of Difficulty: Moderate

    Solution

    The number of electrons participating in the metallic bonding is three for all the lanthanoid elements

    except Eu (At. no. 63) and Yb (At. no. 70) as the outermost electrons are favored to participate in

    bonding. In these two cases, one electron from 5d orbital is shifted to 4f orbital to get half-filled and

    fully filled stable electronic configuration. Hence, these two elements use only two electrons in metallic

    bonding, and atomic radii do not show the expected decrease.

    Correct Option: (4)

  • 8/12/2019 JEE Knocout Test QP_Solution_Dec22 (1)

    61/76

    Question 7:Identify the correct designation for this tripeptide:

    (1) Cys-Met-Glu (2) Cys-Met-Asp (3) Met-Cys-Glu (4) Asp-Cys-Met

    Chapter: Biomolecules

    Level of Difficulty: Easy

    Solution

    Based on the respective structures of amino acids, the correct designation is Cys-Met-Glu.

    Correct Option: (1)

    Question 8:Which of the following ketones has the largest equilibrium constant for the addition of

    water?

    (1) (2)

    (3) (4)

    Chapter:Aldehydes and Ketones

    Level of Difficulty: Hard

    Solution

    The equilibrium constant for hydration depends on stability of produced hydrate and ketone. Since the

    stability of ketone increases with the increasing electron density on carbonyl carbon atom, its stability is

    increased by electron donating group.

    In contrast, the hydrates stability is increased by electron withdrawing group, but stability of ketone is

    decreased.

  • 8/12/2019 JEE Knocout Test QP_Solution_Dec22 (1)

    62/76

  • 8/12/2019 JEE Knocout Test QP_Solution_Dec22 (1)

    63/76

    placing triple bond at carbonyl carbon. In this way, we cannot place a triple bond in the compound given

    in option (3). So, this ketone cannot be formed by alkynes on hydration.

    Correct Option: (3)

    Quick Tip:Just look for minimum two hydrogen atoms at alpha position. If they exist, then alkyne

    formation is possible with the same skeleton and we can also form a ketone from it.

    Question 10:Potassium phthalimide on reaction with compound (A) followed by hydrolysis forms

    isopentyl amine. The compound A will be

    (1) 3 3 3CH CH(CH ) CH(Br) CH (2) 3 3 2 3CH C(CH )(Br) CH CH

    (3) 2 3 2 3Br CH CH(CH ) CH CH (4) 3 3 2 2CH CH(CH ) CH CH Br

    Chapter:Alkyl and Aryl halides

    Level of Difficulty: Moderate

    Solution

    The given reaction sequence is for Gabriel phthalimide synthesis. The synthesis is successful only if

    haloalkane is primary. Secondary haloalkanes produce elimination product, alkenes.

    RNH2is iso-pentyl amine,

    So A will be

    Correct Option: (4)

    Quick Tip:TheNH2group will be attached to that carbon atom at which B atom is present.

    Question 11:Which of the following molecules would you expect to be aromatic?

    NK + RBr

    O

    O

    + H O/H2

    +

    KBr

    C OH

    C OH

    O

    O

    + RNH2

    (A)

    CH3

    CH CH2

    CH2

    NH2

    CH3

    CH3

    CH CH2

    CH2

    Br

    CH3

  • 8/12/2019 JEE Knocout Test QP_Solution_Dec22 (1)

    64/76

    (1) I, II and III only (2) I, II, IV only (3) II, III, IV only (4) All are aromatic

    Chapter: Hydrocarbons: Aromatic

    Level of Difficulty: Moderate

    Solution

    According to Huckels rule, two conditions should satisfy for aromaticity. The planar cyclic compound

    with (4n+ 2) pi electrons are aromatic compounds.

    I. This carbon ring contains 6 pi electrons with all carbon atoms in sp2hybridized state. The angle strain

    in this ring is not much so it is also planar and aromatic.

    II. Though nitrogen atoms can avail lone pairs for aromaticity, but in this case it is not required. Ring

    contains 6 pi electrons and all atoms in sp2

    hybridized state. So, the compound is aromatic.

    III. It contains 8 pi electrons, so it is not an aromatic compound.

    IV. Ring contains 6 pi electrons with all carbon atoms sp2hybridized, so it is an aromatic compound.

    Correct Option: (2)

    Question 12:Which of the following is a copolymer?

    (1)

    (2)

    (3)

  • 8/12/2019 JEE Knocout Test QP_Solution_Dec22 (1)

    65/76

    (4)

    Chapter: Polymers

    Level of Difficulty: Easy

    Solution

    Homopolymers are formed by polymerization of the same type of monomers. Copolymers are formed

    when a mixture of more than one monomeric species is allowed to polymerize. For example, Buna-N is a

    copolymer of acrylonitrile and 1,3-butadiene. From the given options, option (2) represents a

    copolymer.

    Correct Option: (2)

    Question 13:Which of the following carbocations will not rearrange?

    (1) (2) (3) (4)

    Chapter: Some Basic Principles in Organic Chemistry

    Level of Difficulty: Moderate

    Solution

    The rearrangement of a carbocation is possible only if it can form more stable carbocation. Sometimes,

    it can also release angle strain present in relatively smaller or bigger rings. Hence, (1), (2) and (4)

    rearrange as shown below. The compound in (3) cannot rearrange because more stable carbocation is

    not possible by shifting alkyl or halide group.

    (1) (2)

  • 8/12/2019 JEE Knocout Test QP_Solution_Dec22 (1)

    66/76

    (4)

    Correct Option: (3)

    Question 14: The correct order of stability of the following free radicals is

    (1) III > I > II > IV (2) I > II > IV > III (3) IV > II > I > III (4) I > III > II > IV

    Chapter: Some Basic Principles in Organic Chemistry

    Level of Difficulty: Easy

    Solution

    The free radicals are electron-deficient species, whose stability is enhanced by electron donating groups.

    Therefore in (III), allylic free radical is most stable since electron density can be transferred by both

    inductive as well as resonance effects. As (I), (II) and (IV) are 3, 2and 1free radicals, respectively, the

    correct order of stability is III > I > II > IV.

    Correct Option: (1)

    Question 15: Which of the following sets of quantum numbers is not representing the electrons which

    are eliminated from Fe to convert it into Fe3+

    ?

    (1) n= 4, l= 0, m= 0, s= 1/2 (2) n= 4, l= 0, m= 0, s= 1/2

    (3) n= 3, l= 2, m= 0, s= 1/2 (4) n= 3, l= 1, m= 0, s= 1/2

    Chapter:Atomic Structure

    Level of Difficulty: Moderate

    Solution

    Ground state electronic configuration of Fe: 1s22s

    22p

    63s

    23p

    63d

    64s

    2

    The electronic configuration of Fe3+: 1s22s

    22p63s

    23p63d

    54s0

    Hence, two electrons are removed from the fourth shell and the third shell. Two types of orbitals (dand

    s) are involved. The value of lfor s-orbital is 0 and for d-orbital, l = 2.

  • 8/12/2019 JEE Knocout Test QP_Solution_Dec22 (1)

    67/76

    Fourth set of quantum numbers contains l= 1, that is,p-orbital. Thep-orbitals are not involved in

    ionization.

    Correct Option: (4)

    Quick Tip: Just look for the electrons removed from the atom, and from which type orbitals are involved

    in this process.

    Question 16: For which of the following equilibrium, the value of Kpis not greater than the value of KC?

    (1)2

    2C(s) O (g) 2CO(g) (2)2 4 22C(s) 2H O(g) CH (g) CO (g)

    (3)4 2 4 2CuSO 5H O(s) CuSO (s) 5H O(g) (4) 2 2Mg(OH) (s) MgO(s) H O(g)

    Chapter:Chemical Equilibrium

    Level of Difficulty: Easy

    Solution

    ( ) np C

    K K RT ; where n= product side gaseous moles reactant side gaseous moles.

    (1) 2 1 1 ( )p Cn K K RT (2)02 2 0 ( )

    p C p Cn K K RT K K

    (3) 55 0 5 ( )p Cn K K RT (4) 1 0 1 ( )p Cn K K RT

    Correct Option: (2)

    Question 17: The correct order of increasing covalent character of the following is

    (1) KCl < CaCl2< AlCl3< SiCl4 (2) SiCl4< AlCl3< CaCl2< KCl

    (3) AlCl3< CaCl2< KCl < SiCl4 (4) CaCl2< SiCl4< KCl < AlCl3

    Chapter:Chemical Bonding and Molecular Structure

    Level of Difficulty: Easy

    Solution

    According to Fajans rule, as the charge on the cation increases, its effective nuclear charge as well as

    polarizing power increases. Hence, covalent character will also increase. The increasing order should be

    KCl < CaCl2< AlCl3< SiCl4.

    Correct Option: (1)

    Question 18: The correct order of increasing CO bond length of2

    3 2CO, CO , CO

    is

    (1)2

    2 3CO CO CO (2) 23 2CO CO CO

  • 8/12/2019 JEE Knocout Test QP_Solution_Dec22 (1)

    68/76

    (3)2

    2 3CO CO CO (4) 23 2CO CO CO

    Chapter:Chemical Bonding and Molecular Structure

    Level of Difficulty: Moderate

    Solution

    All bonds in 23CO are equivalent because it is resonance hybrid.

    So, the bond order of 23CO is 4/3.

    Bond order in CO is 3 and of CO2it is 2.

    Since bond length is inversely proportional to the bond order, bond length order is2

    2 3CO CO CO

    Correct Option:(1)

    Question 19: The reaction of tin metal with acid can be written as

    2+

    2 o

    2 Sn /SnSn(s) 2H (aq) Sn (aq) H (g); 0.14 VE

    Assume that2+[Sn ] 1M and the partial pressure of hydrogen gas is 1 atm, then the

    (1) cell reaction is spontaneous at pH = 5.

    (2) cell reaction is non-spontaneous at standard conditions.

    (3) cell reaction is spontaneous at pH = 2.

    (4) cell reaction is spontaneous for all pH values.

    Chapter:Electrochemistry

    Level of Difficulty: Hard

    Solution

    2

    2Sn(s) 2H (aq) Sn (aq) H (g)

    Nernst equation for this cell reaction: 22

    Ho

    cell cell 2

    [Sn ]ln

    [H ]

    pRTE E

    nF

    At standard temperature; 22

    Ho

    cell cell 2

    [Sn ]0.0591ln

    [H ]

    pE E

    n

    + 2+2

    o o o

    cell H /H Sn /Sn 0 ( 0.14 V) 0.14 VE E E

    C

    O O

    O

    C

    O O

    O

    C

    O O

    O

  • 8/12/2019 JEE Knocout Test QP_Solution_Dec22 (1)

    69/76

    For pH = 5,cell 5 2

    0.059 10.14 log 0.16 V

    2 (10 )E

    For standard state;o

    cell cell 0.14 VE E

    For pH = 2, cell 2 20.059 1

    0.14 log 0.14 0.12 0.02 V2 (10 )E

    Correct Option: (3)

    Question 20: Select the correct statement about compressibility factor of one mole gas which is kept

    inside a 22.4 L vessel at 273 K.

    (1) ForZ> 1 the pressure of gas will be less than 1 atm.

    (2) ForZ> 1 the pressure of gas will be more than 1 atm.

    (3) If b dominates, pressure will be less than 1 atm.(4) If a dominates, pressure will be greater than 1 atm.

    Chapter:Gaseous State

    Level of Difficulty: Hard

    Solution

    Using ideal gas law:1 0.082 273

    1atm22.4

    nRTp

    V

    So if the gas is ideal, then its pressure is 1 atm. However, it is a real gas, so at different conditions its

    pressure may vary from the ideal pressure.

    If its compressibility factorZ> 1, this indicates that repulsive forces dominate inside the gas. These

    repulsive forces imply that the pressure should be more than ideal pressure, that is, more than 1 atm

    pressure.

    If b dominates, then mm( ) 1pV pb

    p V b RT ZRT RT

    This equation implies thatZwill be more than one always, and pressure should be more than 1 atm.

    If a dominates, then

    mm2

    m m

    1pVa a

    p V RT ZV RT RTV

    This equation implies thatZ< 1, so the pressure should be less than 1 atm as the attraction forces will

    dominate here.

    Correct Option: (2)

  • 8/12/2019 JEE Knocout Test QP_Solution_Dec22 (1)

    70/76

    Question 21: The precipitate of CaF2(Ksp= 1.7 1010

    ) is observed when equal volumes of the following

    are mixed.

    (1)4 2+ 4

    10 M Ca + 10 M F (2)2 2+ 3

    10 M Ca + 10 M F

    (3)5 2+ 3

    10 M Ca + 10 M F (4)3 2+ 5

    10 M Ca + 10 M F

    Chapter:Ionic Equilibrium

    Level of Difficulty: Moderate

    Solution

    The reaction is

    2

    2CaF Ca 2F

    The solubility product is 2 2 10sp [Ca ][F ] 1.7 10K

    The ionic product is2 2

    I [Ca ][F ]K

    (1)2 2 4 4 2 13

    I [Ca ][F ] 0.5 10 (0.5 10 ) 1.25 10K

    (2)2 2 2 3 2 9

    I [Ca ][F ] 0.5 10 (0.5 10 ) 1.25 10K

    (3)2 2 5 3 2 12

    I [Ca ][F ] 0.5 10 (0.5 10 ) 1.25 10K

    (4)2 2 3 5 2 14

    I [Ca ][F ] 0.5 10 (0.5 10 ) 1.25 10K

    In the case of (2), ionic product is greater than solubility product, so precipitation will be observed in this

    case.

    Correct Option: (2)

    Question 22: Consider the decomposition of gaseous N2O5; 2 5 2 22N O 4NO (g) O (g) . At moderate

    gas pressures, it follows the mechanism given below:

    1

    1

    2 5 2 3N O NO +NOk

    k

    2

    2 3 2 2

    NO NO NO NO Ok

    3

    3 2NO NO 2NOk

    Which of the following statements is correct?

    (1) Overall order of the reaction is 2.0 (2) Order w.r.t. N2O5is 1.0

    (3) Order w.r.t. NO3is 1.0 (4) Order cannot be predicted

    Chapter: Chemical Kinetics

  • 8/12/2019 JEE Knocout Test QP_Solution_Dec22 (1)

    71/76

    Level of Difficulty: Hard

    Solution

    The molecule NO3is an intermediate whose concentration is small and can be calculated by the steady

    state approximation. Same can be said for NO.

    Rate of production of NO = Rate of destruction of NO

    22 2 3 3 3 2

    3

    [NO ][NO ] [NO][NO ] [NO] [NO ]k

    k kk

    For NO3, rate of production of NO3= rate of destruction of NO3

    1 2 5 1 2 2 2 3 3[N O ] ( [NO ] [NO ] [NO])[NO ]k k k k 1 2 5

    3

    1 2 2 2

    [N O ][NO ]

    [NO ] 2 [NO ]

    k

    k k

    for the steady state condition of NO3.The rate of production of oxygen, which is also the rate of reaction,

    is

    22 2 3

    d[O ][NO ][NO ]

    dk

    t

    Substituting the value of [NO3], we get

    1 2 2 522 5

    1 2

    [N O ]d[O ][N O ]

    d 2

    k kk

    t k k

    where 1 2

    1 22

    k kk

    k k

    .

    Correct Option: (2)

    Question 23: Which of the following statements is correct about the defects in the crystals?

    (1) Schottky defects are observed in the compounds in which there is a large difference in the size of

    cation and anion, whereas Frenkel is observed when cation and anion are approximately of the same

    size.

    (2) Schottky defects disturb the ratio of cations and anions, whereas Frenkel defects maintain the ratio.

    (3) Schottky defects lead to lowering in density, whereas Frenkel defects do not affect the density.

    (4) Both interstitial defects and Frenkel defects cause increase in the density of solid.

    Chapter: Solid State

    Level of Difficulty: Moderate

    Solution

  • 8/12/2019 JEE Knocout Test QP_Solution_Dec22 (1)

    72/76

    Schottky defect is observed in ionic solids in which cationic and anionic sizes are approximately the

    same. It arises when equal number of cations and anions are missing from their position. It causes

    decrease in the density of solid.

    Frenkel defect is observed in ionic solids in which cationic and anionic size difference is large. It arises

    when the smaller ion leaves its original place and moves into interstitial spaces. This causes no change in

    cationanion ratio, and hence no change in the density.

    Interstitial defect arises when a constituting particle is also placed into interstitial space. This causes an

    increase in density.

    Correct Option: (3)

    Question 24: The solubility of pure oxygen in water at 20C and 1 atm pressure is 43 mg O2per liter of

    water. When air is in contact with water and the air pressure is 585 torr at 20C, how much amount of

    oxygen is dissolved in 500 ml water? The average concentration of oxygen in air is 21.1%(V/V).

    (1) 7 mg (2) 3.5 mg (3) 16.5 mg (4) 33 mg

    Chapter:Solutions

    Level of Difficulty: Moderate

    Solution

    From Henrys law: 1 1gas H gas2 2

    C pC K p

    C p

    Given that C1= 43 mg/l;p1= 1 atm = 760 torr; C2= ?;p2=21.1

    585 123.435 torr 100

    Substituting in the above expression, we get

    2

    123.44 43mg/L 7 mg/L

    760C

    Therefore,2

    7Mass of O in 500 ml water 500 3.5 mg

    1000

    Correct Option: (2)

    Question 25: A saturated solution is prepared by dissolving 0.2 g of polypeptide in water to give 500 mL

    of solution. The solution has an osmotic pressure of 3.74 torr at 27C. What is the approximate

    molecular mass of the polypeptide?

    (1) 8559 (2) 2000 (3) 180 (4) 203

    Chapter:Solutions

  • 8/12/2019 JEE Knocout Test QP_Solution_Dec22 (1)

    73/76

  • 8/12/2019 JEE Knocout Test QP_Solution_Dec22 (1)

    74/76

    We have o o og g

    H U n RT U H n RT

    Moles of N2O taken =176

    4 mol44

    Therefore,oMoles Change for one mole 4 ( )gU H n RT

    or8.314 300

    4 163 1 kJ 662 kJ1000

    U

    Correct Option: (1)

    Question 28: Which of the following statements is not correct?

    (1) Final temperature in reversible adiabatic expansion is greater than that in irreversible adiabatic

    expansion.

    (2) When heat is supplied to an ideal gas in isothermal process, kinetic energy of gas remains constant.

    (3) When an ideal gas is subjected to adiabatic expansion, it gets cooled.

    (4) Entropy increases when an ideal gas expands isothermally.

    Chapter:Chemical Thermodynamics

    Level of Difficulty: Hard

    Solution

    For adiabatic process, VU w w C T V

    wT

    C

    Since work done in reversible process is more than that in irreversible process, the temperature change

    is more in reversible process, that is, the temperature decrease will be more.

    Since kinetic energy of ideal gases is proportional to root of temperature, for isothermal it will not

    change.

    From the above relations, it is clear that when an ideal gas is subjected to adiabatic expansion, its

    temperature decreases.

    Entropy increases when an ideal gas expands isothermally because increasing the volume causes

    increase in randomness.Correct Option: (1)

    Question 29: Ammonium nitrate detonates above 300C according to the chemical equation given

    below:

    4 3 2 2 2NH NO N O H O

  • 8/12/2019 JEE Knocout Test QP_Solution_Dec22 (1)

    75/76

    Select the correct statement:

    (1) The volume change is approximately 3.5 times the initial volume at STP.

    (2) The volume change is approximately 1.5 times the initial volume taken at 323C.

    (3) On explosion of 80 g of ammonium nitrate, 78.4 L of total volume is obtained at STP.

    (4) 100 g of NH4NO3produces volume of 215.3 L of the total gases at 323C and 1 atm pressure.

    Chapter: Some Basic Concepts of Chemistry

    Level of Difficulty: Moderate

    Solution

    The balanced form of equation is:4 3 2 2 22NH NO (s) 2N (g) O (g) 4H O(g)

    Since the reactant is in solid state, so the volume occupied by it is negligible.

    The moles are approximately 3.5 times, but the volume is not 3.5 times. At STP, water is l iquefied so

    volume is reduced and corresponds to the gaseous moles left.

    Taking 80 g of NH4NO3(one mole) will produces 3.5 mol

    But at STP, gaseous moles are 1.5 mol, so volume produced is approximately 1.5 22.4 L 33.6 L

    100 g of NH4NO3contains100

    1.25 mol80

    Therefore, gaseous moles appeared at given condition 3.5 1.25 4.375 mol

    The volume of gases,3.5 1.25 0.082 600

    215.3 L1

    nRTV

    p

    Correct Option: (4)

    Question 30:The presence of halogen, in an organic compound, is detected by

    (1) iodoform test. (2) Tollens reagent. (3) Beilsteins test. (4) Millons test.

    Chapter:Purification and Characterization of Organic Compounds

    Level of Difficulty: Easy

    Solution

    In Beilstein test, a (clean) copper wire is heated in the non-luminous flame of the Bunsen burner until it

    ceases to impart any green or bluish green color to the flame. The heated end of the wire is dipped into

    the organic compound and is again introduced into the Bunsen flame. The appearance of a bluish green

    or green flame due to the formation of volatile cupric halides shows the presence of halogens in the

    given organic compound.

    Correct Option:(3).

  • 8/12/2019 JEE Knocout Test QP_Solution_Dec22 (1)

    76/76

    Quick Tip:Compounds like urea, thiourea that do not contain halogen give positive Beilsteins test.

    Fluoride ion is not detected by this test as copper fluoride is not volatile. Hence, it is not a very

    dependable test, and thus, negative Beilstein test is more useful than a positive one.